Exam4Training

BCS CTFL4 ISTQB Certified Tester Foundation Level CTFL 4.0 Exam Online Training

Question #1

The tests at the bottom layer of the test pyramid:

  • A . run faster than the tests at the top layer of the pyramid
  • B . cover larger pieces of functionalities than the tests at the top layer of the pyramid
  • C . are defined as ‘Ul Tests’ or ‘End-To-End tests’ in the different models of the pyramid
  • D . are unscripted tests produced by experience-based test techniques

Reveal Solution Hide Solution

Correct Answer: A
A

Explanation:

The tests at the bottom layer of the test pyramid run faster than the tests at the top layer of the pyramid because they are more focused, isolated, and atomic. They usually test individual units or components of the software system, such as classes, methods, or functions. They are also easier to maintain and execute, as they have fewer dependencies and interactions with other parts of the system. The tests at the top layer of the test pyramid, on the other hand, are slower because they cover larger pieces of functionalities, such as user interfaces, workflows, or end-to-end scenarios. They also have more dependencies and interactions with other systems, such as databases, networks, or external services. They are more complex and costly to maintain and execute, as they require more setup and teardown procedures, test data, and test environments.

Reference: ISTQB Certified Tester Foundation Level (CTFL) v4.0 sources and documents:

ISTQB® Certified Tester Foundation Level Syllabus v4.0, Chapter 3.2.1, Test Pyramid1 ISTQB® Glossary of Testing Terms v4.0, Test Pyramid2

Question #2

Test automation allows you to:

  • A . demonstrate the absence of defects
  • B . produce tests that are less subject to human errors
  • C . avoid performing exploratory testing
  • D . increase test process efficiency by facilitating management of defects

Reveal Solution Hide Solution

Correct Answer: B
B

Explanation:

Test automation allows you to produce tests that are less subject to human errors, as they can execute predefined test scripts or test cases with consistent inputs, outputs, and expected results. Test automation can also reduce the manual effort and time required to execute repetitive or tedious tests, such as regression tests, performance tests, or data-driven tests. Test automation does not demonstrate the absence of defects, as it can only verify the expected behavior of the system under test, not the unexpected or unknown behavior. Test automation does not avoid performing exploratory testing, as exploratory testing is a valuable technique to discover new information, risks, or defects that are not covered by automated tests. Test automation does not increase test process efficiency by facilitating management of defects, as defect management is a separate activity that involves reporting, tracking, analyzing, and resolving defects, which may or may not be related to automated tests.

Reference: ISTQB Certified Tester Foundation Level (CTFL) v4.0 sources and documents:

ISTQB® Certified Tester Foundation Level Syllabus v4.0, Chapter 3.3.1, Test Automation1 ISTQB® Glossary of Testing Terms v4.0, Test Automation2

Question #3

Which of the following statements about how different types of test tools support testers is true?

  • A . The support offered by a test data preparation tool is often leveraged by testers to run automated regression test suites
  • B . The support offered by a performance testing tool is often leveraged by testers to run load tests
  • C . The support offered by a bug prediction tool is often used by testers to track the bugs they found
  • D . The support offered by a continuous integration tool is often leveraged by testers to automatically generate test cases from a model

Reveal Solution Hide Solution

Correct Answer: B
B

Explanation:

The support offered by a performance testing tool is often leveraged by testers to run load tests, which are tests that simulate a large number of concurrent users or transactions on the system under test, in order to measure its performance, reliability, and scalability. Performance testing tools can help testers to generate realistic workloads, monitor system behavior, collect and analyze performance metrics, and identify performance bottlenecks.

The other statements are false, because:

A test data preparation tool is a tool that helps testers to create, manage, and manipulate test data, which are the inputs and outputs of test cases. Test data preparation tools are not directly related to running automated regression test suites, which are test suites that verify that the system still works as expected after changes or modifications. Regression test suites are usually executed by test execution tools, which are tools that can automatically run test cases and compare actual results with expected results.

A bug prediction tool is a tool that uses machine learning or statistical techniques to predict the likelihood of defects in a software system, based on various factors such as code complexity, code churn, code coverage, code smells, etc. Bug prediction tools are not used by testers to track the bugs they found, which are the actual defects that have been detected and reported during testing. Bugs are usually tracked by defect management tools, which are tools that help testers to record, monitor, analyze, and resolve defects.

A continuous integration tool is a tool that enables the integration of code changes from multiple

developers into a shared repository, and the execution of automated builds and tests, in order to ensure the quality and consistency of the software system. Continuous integration tools are not used by testers to automatically generate test cases from a model, which are test cases that are derived from a representation of the system under test, such as a state diagram, a decision table, a use case, etc. Test cases can be automatically generated by test design tools, which are tools that support the implementation and maintenance of test cases, based on test design specifications or test models.

Reference: ISTQB Certified Tester Foundation Level (CTFL) v4.0 sources and documents: ISTQB® Certified Tester Foundation Level Syllabus v4.0, Chapter 3.4.1, Types of Test Tools

ISTQB® Glossary of Testing Terms v4.0, Performance Testing Tool, Test Data Preparation Tool, Bug Prediction Tool, Continuous Integration Tool, Test Execution Tool, Defect Management Tool, Test Design Tool

Question #4

Which of the following statements about branch coverage is true?

  • A . The minimum number of test cases needed to achieve full branch coverage, is usually lower than that needed to achieve full statement coverage
  • B . If full branch coverage has been achieved, then all unconditional branches within the code have surely been exercised
  • C . If full branch coverage has been achieved, then all combinations of conditions in a decision table have surely been exercised
  • D . Exercising at least one of the decision outcomes for all decisions within the code, ensures achieving full branch coverage

Reveal Solution Hide Solution

Correct Answer: D
D

Explanation:

Exercising at least one of the decision outcomes for all decisions within the code, ensures achieving full branch coverage, which is a test coverage criterion that requires that all branches in the control flow of the code are executed at least once by the test cases. A branch is a basic block of code that has a single entry point and a single exit point, and a decision is a point in the code where the control flow can take more than one direction, such as an if-then-else statement, a switch-case statement, a loop statement, etc. The decision outcomes are the possible paths that can be taken from a decision, such as the then branch or the else branch, the case branch or the default branch, the loop body or the loop exit, etc. The other statements are false, because:

The minimum number of test cases needed to achieve full branch coverage, is usually higher than that needed to achieve full statement coverage, which is a test coverage criterion that requires that all executable statements in the code are executed at least once by the test cases. This is because branch coverage is a stronger criterion than statement coverage, as it implies statement coverage, but not vice versa. For example, a single test case can achieve full statement coverage for an if-then-else statement, but two test cases are needed to achieve full branch coverage, as both the then branch and the else branch need to be exercised.

If full branch coverage has been achieved, then all unconditional branches within the code have not necessarily been exercised, as unconditional branches are branches that do not depend on any decision, and are always executed, such as a goto statement, a break statement, a return statement, etc. Unconditional branches are not part of the branch coverage criterion, as they do not represent different paths in the control flow of the code. However, they are part of the statement coverage criterion, as they are executable statements in the code.

If full branch coverage has been achieved, then all combinations of conditions in a decision table have not necessarily been exercised, as a decision table is a test design technique that represents the logical relationships between multiple conditions and their corresponding actions, in a tabular format. A decision table can have more combinations of conditions than the number of decision outcomes in the code, as each condition can have two or more possible values, such as true or false, yes or no, etc. For example, a decision table with four conditions can have 16 combinations of conditions, but the corresponding code may have only two decision outcomes, such as pass or fail. To exercise all combinations of conditions in a decision table, a stronger test coverage criterion is needed, such as condition combination coverage, which requires that all possible combinations of condition outcomes in the code are executed at least once by the test cases.

Reference: ISTQB Certified Tester Foundation Level (CTFL) v4.0 sources and documents:

ISTQB® Certified Tester Foundation Level Syllabus v4.0, Chapter 2.3.1, Test Coverage Criteria Based on the Structure of the Software

ISTQB® Glossary of Testing Terms v4.0, Branch Coverage, Statement Coverage, Branch, Decision, Decision Outcome, Unconditional Branch, Decision Table, Condition Combination Coverage

Question #5

Consider the following simplified version of a state transition diagram that specifies the behavior of a video poker game:

What Is the minimum number of test cases needed to cover every unique sequence of up to 3 states/2 transitions starting In the "Start" state and ending In the "End" state?

  • A . 1
  • B . 2
  • C . 3
  • D . 4

Reveal Solution Hide Solution

Correct Answer: D
D

Explanation:

The minimum number of test cases needed to cover every unique sequence of up to 3 states/2 transitions starting in the “Start” state and ending in the “End” state is 4. This is because there are 4 unique sequences of up to 3 states/2 transitions starting in the “Start” state and ending in the “End” state:

Start -> Bet -> End

Start -> Deal -> End

Start -> 1st Deal -> End

Start -> 2nd Deal -> End

Reference: ISTQB Certified Tester Foundation Level (CTFL) v4.0 sources and documents.

Question #6

Consider a given test plan which, among others, contains the following three sections: "Test Scope", "Testing Communication", and "Stakeholders".

The features of the test object to be tested and those excluded from the testing represent information that is:

  • A . not usually included in a test plan, and therefore in the given test plan it should not be specified neither within the three sections mentioned, nor within the others
  • B . usually included in a test plan and, in the given test plan, it is more likely to be specified within "Test Scope" rather than in the other two sections mentioned
  • C . usually included in a test plan and, in the given test plan, it is more likely to be specified within "Testing Communication" rather than in the other two sections mentioned
  • D . usually included in a test plan and, in the given test plan, it is more likely to be specified within "Stakeholders" rather than in the other two sections mentioned

Reveal Solution Hide Solution

Correct Answer: B
B

Explanation:

The features of the test object to be tested and those excluded from the testing represent information that is usually included in a test plan and, in the given test plan, it is more likely to be specified within “Test Scope” rather than in the other two sections mentioned. The test scope defines the boundaries and limitations of the testing activities, such as the test items, the features to be tested, the features not to be tested, the test objectives, the test environment, the test resources, the test assumptions, the test risks, etc. The test scope helps to establish a common understanding of what is included and excluded from the testing, and to avoid ambiguity, confusion, or misunderstanding among the stakeholders. The other two sections, “Testing Communication” and “Stakeholders”, are also important parts of a test plan, but they do not directly address the features of the test object. The testing communication describes the methods, frequency, and responsibilities for the communication and reporting of the testing progress, status, issues, and results. The stakeholders identify the roles and responsibilities of the people involved in or affected by the testing activities, such as the test manager, the test team, the project manager, the developers, the customers, the users, etc.

Reference: ISTQB Certified Tester Foundation Level (CTFL) v4.0 sources and documents:

ISTQB® Certified Tester Foundation Level Syllabus v4.0, Chapter 2.1.1, Test Planning1 ISTQB® Glossary of Testing Terms v4.0, Test Plan, Test Scope2

Question #7

An alphanumeric password must be between 4 and 7 characters long and must contain at least one numeric character, one capital (uppercase) letter and one lowercase letter of the alphabet.

Which one of the following sets of test cases represents the correct outcome of a two-value boundary value analysis applied to the password length? (Note: test cases are separated by a semicolon)

  • A . 1xA;aB11;Pq1ZZab;7iDD0a1x
  • B . aB11;99rSp:5NnN10;7iDD0a1x
  • C . 1xB: aB11: 99rSp: 5NnN10; 4NnN10T; 44ghWn19
  • D . 1RhT;rSp53;3N3e10;8sBdby

Reveal Solution Hide Solution

Correct Answer: D
D

Explanation:

The correct outcome of a two-value boundary value analysis applied to the password length is the set of test cases represented by option D. Boundary value analysis is a test design technique that focuses on the values at the boundaries of an equivalence partition, such as the minimum and maximum values, or the values just above and below the boundaries. A two-value boundary value analysis uses two values for each boundary, one representing the valid value and one representing the invalid value.

For example, if the valid range of values is from 4 to 7, then the two values for the lower boundary are 3 and 4, and the two values for the upper boundary are 7 and 8. The test cases in option D use these values for the password length, while also satisfying the other requirements of the password, such as containing at least one numeric character, one capital letter, and one lowercase letter.

The test cases in option D are:

1RhT: a 4-character password that is valid

rSp53: a 5-character password that is valid

3N3e10: a 6-character password that is valid

8sBdby: an 8-character password that is invalid The test cases in the other options are incorrect, because they either use values that are not at the boundaries of the password length, or they do not meet the other requirements of the password. For example, the test cases in option A are: 1xA: a 3-character password that is invalid, but it does not contain a capital letter

aB11: a 4-character password that is valid

Pq1ZZab: a 7-character password that is valid

7iDD0a1x: an 8-character password that is invalid

Reference: ISTQB Certified Tester Foundation Level (CTFL) v4.0 sources and documents:

ISTQB® Certified Tester Foundation Level Syllabus v4.0, Chapter 2.2.1, Black-box Test Design

Techniques1

ISTQB® Glossary of Testing Terms v4.0, Boundary Value Analysis, Equivalence Partition2

Question #8

A company wants to reward each of its salespeople with an annual bonus that represents the sum of all the bonuses accumulated for every single sale made by that salesperson. The bonus for a single sale can take on the following four values: 3%, 5%, 7% and 10% (the percentage refers to the amount of the single sale). These values are determined on the basis of the type of customer (classified as "Basic" or "Premium") to which such sale was made, and on the amount of such sale classified into the following three groups G1, G2 and G3:

• [G1]: less than 300 euros

• [G2]: between 300 and 2000 euros

• [G3]: greater than 2000 euros

Which of the following is the minimum number of test cases needed to cover the full decision table associated with this scenario?

  • A . 12
  • B . 6
  • C . 4
  • D . 3

Reveal Solution Hide Solution

Correct Answer: B
B

Explanation:

The minimum number of test cases needed to cover the full decision table associated with this scenario is 6. This is because the decision table has 4 conditions (type of customer and amount of sale) and 4 actions (bonus percentage). The conditions have 2 possible values each (Basic or Premium, and G1, G2 or G3), so the total number of combinations is 2 x 2 x 2 x 2 = 16. However, not all combinations are valid, as some of them are contradictory or impossible. For example, a sale cannot be both less than 300 euros and greater than 2000 euros at the same time. Therefore, we need to eliminate the invalid combinations and keep only the valid ones.

The valid combinations are:

These 6 combinations cover all the possible values of the conditions and actions, and they are the minimum number of test cases needed to cover the full decision table.

Reference: ISTQB Certified Tester Foundation Level (CTFL) v4.0 sources and documents,

Question #9

Which of the following statements is true?

  • A . Experience-based test techniques rely on the experience of testers to identify the root causes of defects found by black-box test techniques
  • B . Some of the most common test basis used by white-box test techniques include user stories, use cases and business processes
  • C . Experience-based test techniques are often useful to detect hidden defects that have not been targeted by black-box test techniques
  • D . The primary goal of experience-based test techniques is to design test cases that can be easily automated using a GUI-based test automation tool

Reveal Solution Hide Solution

Correct Answer: C
C

Explanation:

Experience-based test techniques are test design techniques that rely on the experience, knowledge, intuition, and creativity of the testers to identify and execute test cases that are likely to find defects in the software system. Experience-based test techniques are often useful to detect hidden defects that have not been targeted by black-box test techniques, which are test design techniques that use the external behavior and specifications of the software system as the test basis, without considering its internal structure or implementation. Experience-based test techniques can complement black-box test techniques by covering aspects that are not explicitly specified, such as usability, security, reliability, performance, etc.

The other statements are false, because:

Experience-based test techniques do not rely on the experience of testers to identify the root causes of defects found by black-box test techniques, but rather to identify the potential sources of defects based on their own insights, heuristics, or exploratory testing. The root causes of defects are usually identified by debugging or root cause analysis, which are activities that involve examining the code or the development process to find and fix the errors that led to the defects.

Some of the most common test basis used by white-box test techniques include the source code, the design documents, the architecture diagrams, and the control flow graphs of the software system. White-box test techniques are test design techniques that use the internal structure and implementation of the software system as the test basis, and aim to achieve a certain level of test coverage based on the code elements, such as statements, branches, paths, etc. User stories, use cases, and business processes are examples of test basis used by black-box test techniques, as they describe the functional and non-functional requirements of the software system from the perspective of the users or the stakeholders.

The primary goal of experience-based test techniques is not to design test cases that can be easily automated using a GUI-based test automation tool, but rather to design test cases that can reveal defects that are not easily detected by other test techniques, such as boundary value analysis, equivalence partitioning, state transition testing, etc. Test automation is the use of software tools to execute test cases and compare actual results with expected results, without human intervention. Test automation can be applied to different types of test techniques, depending on the test objectives, the test levels, the test tools, and the test resources. However, test automation is not always feasible or beneficial, especially for test cases that require human judgment, creativity, or exploration, such as those designed by experience-based test techniques.

Reference: ISTQB Certified

Tester Foundation Level (CTFL) v4.0 sources and documents:

ISTQB® Certified Tester Foundation Level Syllabus v4.0, Chapter 2.2.1, Black-box Test Design Techniques

ISTQB® Certified Tester Foundation Level Syllabus v4.0, Chapter 2.2.2, White-box Test Design Techniques

ISTQB® Certified Tester Foundation Level Syllabus v4.0, Chapter 2.2.3, Experience-based Test Design Techniques

ISTQB® Glossary of Testing Terms v4.0, Experience-based Test Technique, Black-box Test Technique, White-box Test Technique, Test Basis, Test Coverage, Test Automation

Question #10

A new web app aims at offering a rich user experience. As a functional tester, you have run some functional tests to verify that, before releasing the app, such app works correctly on several mobile devices, all of which are listed as supported devices within the requirements specification. These tests were performed on stable and isolated test environments where you were the only user interacting with the application. All tests passed, but in some of those tests you observed the following issue: on some mobile devices only, the response time for two web pages containing images was extremely slow.

Based only on the given information, which of the following recommendation would you follow?

  • A . You should open a defect report providing detailed information on which devices and by running which tests you observed the issue
  • B . The issue is related to performance efficiency, not functionality. Thus, as a functional tester, you should not open any defect report as all the functional tests passed
  • C . You should not open any defect report as the problem is most likely due to poor hardware equipment on the devices where you observed the issue
  • D . You should not open any defect report and inform the test manager that the devices on which you observed the issue should no longer be supported so that they will be removed from the requirements specification

Reveal Solution Hide Solution

Correct Answer: A
A

Explanation:

As a functional tester, you should open a defect report providing detailed information on which devices and by running which tests you observed the issue. A defect report is a document that records the occurrence, nature, and status of a defect detected during testing, and provides information for further investigation and resolution. A defect report should include relevant information such as the defect summary, the defect description, the defect severity, the defect priority, the defect status, the defect origin, the defect category, the defect reproduction steps, the defect screenshots, the defect attachments, etc. Opening a defect report is a good practice for any tester who finds a defect in the software system, regardless of the type or level of testing performed.

The other options are not recommended, because:

The issue is related to performance efficiency, not functionality, but that does not mean that as a functional tester, you should not open any defect report as all the functional tests passed. Performance efficiency is a quality characteristic that measures how well the software system performs its functions under stated conditions, such as the response time, the resource utilization, the throughput, etc. Performance efficiency is an important aspect of the user experience, especially for web applications that run on different devices and networks. Even if the functional tests passed, meaning that the software system met the functional requirements, the performance issue observed on some devices could still affect the user satisfaction, the usability, the reliability, and the security of the software system. Therefore, as a functional tester, you have the responsibility to report the performance issue as a defect, and provide as much information as possible to help the developers or the performance testers to investigate and resolve it.

Question #11

For each of the test cases to be executed, the following table specifies the priority order and dependencies on other test cases

Which of the following test execution schedules is compatible with the logical dependencies and allows executing the test cases in priority order?

  • A . TC4, TC3, TC2, TC6, TC5. TC1
  • B . TC4, TC6, TC3, TC2, TC5, TC1
  • C . TC3, TC5, TC6, TC1, TC4, TC3
  • D . TC4, TC3, TC2, TC6, TC1, TC5

Reveal Solution Hide Solution

Correct Answer: D
D

Explanation:

This answer is correct because it follows the logical dependencies and allows executing the test cases in priority order. TC4, TC3, and TC2 are executed first because they have the highest priority. TC6 is executed next because it has a logical dependency on TC2. TC1 is executed next because it has a logical dependency on TC5. Finally, TC5 is executed last because it has the lowest priority.

Reference: ISTQB Certified Tester Foundation Level (CTFL) v4.0 documents

Question #12

Which of the following statements best describes how configuration management supports testing?

  • A . Configuration management helps reduce testing effort by identifying a manageable number of test environment configurations in which to test the software, out of all possible configurations of the environment in which the software will be released
  • B . Configuration management is an administrative discipline that includes change control, which is the process of controlling the changes to identified items referred to as Configuration Items’
  • C . Configuration management is an approach to interoperability testing where tests are executed in the cloud, as the cloud can provide cost-effective access to multiple configurations of the test environments
  • D . Configuration management helps ensure that all relevant project documentation and software items are uniquely identified in all their versions and therefore can be unambiguously referenced in test documentation

Reveal Solution Hide Solution

Correct Answer: D
D

Explanation:

This answer is correct because configuration management is a process of establishing and maintaining consistency of a product’s performance, functional, and physical attributes with its requirements, design, and operational information throughout its life. Configuration management helps ensure that all relevant project documentation and software items are uniquely identified in all their versions and therefore can be unambiguously referenced in test documentation. This supports testing by providing traceability, consistency, and control over the test artifacts and the software under test.

Reference: ISTQB Glossary of Testing Terms v4.0, : ISTQB Foundation Level Syllabus v4.0, Section 2.2.2.2

Question #13

Which of the following statements about exploratory testing is true?

  • A . Exploratory testing is an experience-based test technique in which testers explore the requirements specification to detect non testable requirements
  • B . When exploratory testing is conducted following a session-based approach, the issues detected by the testers can be documented in session sheets
  • C . Exploratory testing is an experience-based test technique used by testers during informal code reviews to find defects by exploring the source code
  • D . In exploratory testing, testers usually produce scripted tests and establish bidirectional traceability between these tests and the items of the test basis

Reveal Solution Hide Solution

Correct Answer: B
B

Explanation:

Exploratory testing is an experience-based test technique in which testers dynamically design and execute tests based on their knowledge, intuition, and learning of the software system, without following predefined test scripts or test cases. Exploratory testing can be conducted following a session-based approach, which is a structured way of managing and measuring exploratory testing. In a session-based approach, the testers perform uninterrupted test sessions, usually lasting between 60 and 120 minutes, with a specific charter or goal, and document the issues detected, the test coverage achieved, and the time spent in session sheets. Session sheets are records of the test activities, results, and observations during a test session, which can be used for reporting, debriefing, and learning purposes.

The other statements are false, because:

Exploratory testing is not a test technique in which testers explore the requirements specification to detect non testable requirements, but rather a test technique in which testers explore the software system to detect functional and non-functional defects, as well as to learn new information, risks, or opportunities. Non testable requirements are requirements that are ambiguous, incomplete, inconsistent, or not verifiable, which can affect the quality and effectiveness of the testing process. Non testable requirements can be detected by applying static testing techniques, such as reviews or inspections, to the requirements specification, before the software system is developed or tested. Exploratory testing is not a test technique used by testers during informal code reviews to find defects by exploring the source code, but rather a test technique used by testers during dynamic testing to find defects by exploring the behavior and performance of the software system, without examining the source code. Informal code reviews are static testing techniques, in which the source code is analyzed by one or more reviewers, without following a formal process or using a checklist, to identify defects, violations, or improvements. Informal code reviews are usually performed by developers or peers, not by testers.

In exploratory testing, testers usually do not produce scripted tests and establish bidirectional traceability between these tests and the items of the test basis, but rather produce unscripted tests and adapt them based on the feedback and the findings of the testing process. Scripted tests are tests that are designed and documented in advance, with predefined inputs, outputs, and expected results, and are executed according to a test plan or a test procedure. Bidirectional traceability is the ability to trace both forward and backward the relationships between the items of the test basis, such as the requirements, the design, the risks, etc., and the test artifacts, such as the test cases, the test results, the defects, etc. Scripted tests and bidirectional traceability are usually associated with more formal and structured testing approaches, such as specification-based or structure-based test techniques, not with exploratory testing.

Reference: ISTQB Certified Tester Foundation Level (CTFL)

v4.0 sources and documents:

ISTQB® Certified Tester Foundation Level Syllabus v4.0, Chapter 2.2.3, Experience-based Test Design Techniques1

ISTQB® Glossary of Testing Terms v4.0, Exploratory Testing, Session-based Testing, Session Sheet, Non Testable Requirement, Static Testing, Informal Review, Dynamic Testing, Scripted Testing, Bidirectional Traceability2

Question #14

In a two-hour uninterrupted test session, performed as part of an iteration on an Agile project, a heuristic checklist was used to help the tester focus on some specific usability issues of a web application.

The unscripted tests produced by the tester’s experience during such session belong to which one of the following testing quadrants?

  • A . Q1
  • B . Q2
  • C . Q3
  • D . Q4

Reveal Solution Hide Solution

Correct Answer: C
C

Explanation:

The unscripted tests produced by the tester’s experience during the two-hour test session belong to the testing quadrant Q3. The testing quadrants are a classification of testing types based on two dimensions: the test objectives (whether the testing is focused on supporting the team or critiquing the product) and the test basis (whether the testing is based on the technology or the business). The testing quadrants are labeled as Q1, Q2, Q3, and Q4, and each quadrant represents a different testing perspective, such as unit testing, acceptance testing, usability testing, or performance testing. The testing quadrant Q3 corresponds to the testing types that have the objective of critiquing the product from the business perspective, such as exploratory testing, usability testing, user acceptance testing, alpha testing, beta testing, etc. The unscripted tests performed by the tester in the given scenario are examples of exploratory testing and usability testing, as they are based on the tester’s experience, intuition, and learning of the web application, and they focus on some specific usability issues, such as the user interface, the user satisfaction, the user feedback, etc.

The other options are incorrect, because:

The testing quadrant Q1 corresponds to the testing types that have the objective of supporting the team from the technology perspective, such as unit testing, component testing, integration testing, system testing, etc. These testing types are usually performed by developers or testers who have access to the source code, the design, the architecture, or the configuration of the software system, and they aim to verify the functionality, the quality, and the reliability of the software system at different levels of integration.

The testing quadrant Q2 corresponds to the testing types that have the objective of supporting the team from the business perspective, such as functional testing, acceptance testing, story testing, scenario testing, etc. These testing types are usually performed by testers or customers who have access to the requirements, the specifications, the user stories, or the business processes of the software system, and they aim to validate that the software system meets the expectations and the needs of the users and the stakeholders.

The testing quadrant Q4 corresponds to the testing types that have the objective of critiquing the product from the technology perspective, such as performance testing, security testing, reliability testing, compatibility testing, etc. These testing types are usually performed by testers or specialists who have access to the tools, the metrics, the standards, or the benchmarks of the software system, and they aim to evaluate the non-functional aspects of the software system, such as the efficiency, the security, the reliability, or the compatibility of the software system under different conditions or environments.

Reference: ISTQB Certified Tester Foundation Level (CTFL) v4.0 sources and documents:

ISTQB® Certified Tester Foundation Level Syllabus v4.0, Chapter 1.3.1, Testing in Software Development Lifecycles

ISTQB® Glossary of Testing Terms v4.0, Testing Quadrant, Exploratory Testing, Usability Testing, Unit Testing, Component Testing, Integration Testing, System Testing, Functional Testing, Acceptance Testing, Story Testing, Scenario Testing, Performance Testing, Security Testing, Reliability Testing, Compatibility Testing

Question #15

Which of the following is not an example of a typical content of a test completion report for a test project?

  • A . The additional effort spent on test execution compared to what was planned
  • B . The unexpected test environment downtime that resulted in slower test execution
  • C . The residual risk level if a risk-based test approach was adopted
  • D . The test procedures of all test cases that have been executed

Reveal Solution Hide Solution

Correct Answer: D
D

Explanation:

This answer is correct because the test procedures of all test cases that have been executed are not a typical content of a test completion report for a test project. A test completion report is a document that summarizes the test activities and results at the end of a test project. It usually includes information such as the test objectives, scope, approach, resources, schedule, results, deviations, issues, risks, lessons learned, and recommendations for improvement. The test procedures of all test cases that have been executed are part of the test documentation, but they are not relevant for the test completion report, as they do not provide a high-level overview of the test project outcomes and performance.

Reference: ISTQB Foundation Level Syllabus v4.0, Section 2.5.3.2

Question #16

Which of the following statements about error guessing is true?

  • A . Error guessing is a system that adopts artificial intelligence to predict whether software components are likely to contain defects or not
  • B . Experienced testers, when applying error guessing, rely on the use of a high-level list of what needs to be tested as a guide to find defects
  • C . Error guessing refers to the ability of a system or component to continue normal operation despite the presence of erroneous inputs
  • D . Experienced testers, when applying error guessing technique, can anticipate where errors, defects and failures have occurred and target their tests at those issues

Reveal Solution Hide Solution

Correct Answer: D
D

Explanation:

This answer is correct because error guessing is a test design technique where the experience and intuition of the tester are used to anticipate where errors, defects and failures have occurred or are likely to occur, and to design test cases to expose them. Error guessing can be based on factors such as the complexity of the system or component, the known or suspected weaknesses of the system or component, the previous history of defects, or the common types of errors in the domain or technology. Error guessing can be used as a complementary technique to other more systematic or formal techniques, or when there is insufficient information or time to apply them.

Reference: ISTQB Glossary of Testing Terms v4.0, ISTQB Foundation Level Syllabus v4.0, Section 2.3.2.5

Question #17

Which of the following is a task the Author is responsible for, as part of a typical formal review?

  • A . Determining the people who will be involved in the review
  • B . Recording the anomalies found during the review meeting
  • C . Identifying potential anomalies in the work product under review
  • D . Fixing the anomalies found in the work product under review

Reveal Solution Hide Solution

Correct Answer: C
C

Explanation:

This answer is correct because identifying potential anomalies in the work product under review is one of the tasks the Author is responsible for, as part of a typical formal review. The Author is the person who creates the work product to be reviewed, such as a requirement specification, a design document, or a test case. The Author’s tasks include preparing the work product for the review, identifying potential anomalies in the work product, and fixing the anomalies found in the work product after the review.

Reference: ISTQB Glossary of Testing Terms v4.0, ISTQB Foundation Level Syllabus v4.0, Section 2.4.2.1

Question #18

The whole-team approach:

  • A . promotes the idea that all team members should have a thorough understanding of test techniques
  • B . is a consensus-based approach that engages the whole team in estimating the user stories
  • C . promotes the idea that all team members should be responsible for the quality of the product
  • D . is mostly adopted in projects aimed at developing safety-critical systems, as it ensures the highest level of testing independence

Reveal Solution Hide Solution

Correct Answer: C
C

Explanation:

This answer is correct because the whole-team approach is a way of working in agile projects where all team members share the responsibility for the quality of the product, and collaborate on delivering value to the customer. The whole-team approach involves testers, developers, business analysts, product owners, and other stakeholders in planning, designing, developing, testing, and delivering the product. The whole-team approach fosters communication, feedback, learning, and continuous improvement within the team.

Reference: ISTQB Glossary of Testing Terms v4.0, ISTQB Foundation Level Syllabus v4.0, Section 3.1.1.1

Question #19

Consider the following user story about the authentication functionality of an e-commerce website: "As a logged-in user, I want to change my current password with a new one, so that I can make my account safer".

The following are some of the acceptance criteria defined for the user story:

[a] After the logged-in user has successfully changed his password, an email confirming the change must be sent to him

[b] To successfully change the password, the logged-in user must enter the current password, enter a new valid password, and finally confirm by pressing the ‘Change Password’ button

[c] To be valid, the new password entered by the logged-in user is not only required to meet the criteria related to the length and type of characters, but must also be different form the last 5 passwords of that user

[d] A dedicated error message must be presented to the logged-in user when he enters a wrong current password

[e] A dedicated error message must be presented to the logged-in user when he enters the correct current password, but enters an invalid password

Based only on the given information, which of the following ATDD tests is most likely to be written first?

  • A . The logged-in user enters a wrong current password and views the dedicated error message
  • B . The logged-in user enters the correct current password, enters a valid new password (different from the last 5 passwords), presses the Change Password’ button, and finally receives the e-mail confirming that the password has been successfully changed
  • C . The logged-in user enters the correct current password, enters an invalid password, and finally views the dedicated error
  • D . The logged-in user submits a purchase order containing ten items, selects to pay with a Visa credit card, enters credit card information of a valid card, presses the ‘Confirm’ button, and finally views the dedicated message confirming that the purchase has been successful

Reveal Solution Hide Solution

Correct Answer: B
B

Explanation:

ATDD stands for Acceptance Test-Driven Development, which is a collaborative approach to software development and testing, in which the acceptance criteria of a user story are defined and automated as executable tests before the implementation of the software system. ATDD tests are usually written in a Given-When-Then format, which describes the preconditions, the actions, and the expected outcomes of a test scenario. ATDD tests are intended to verify that the software system meets the expectations and the needs of the users and the stakeholders, as well as to provide feedback and guidance for the developers and the testers. Based on the given information, the ATDD test that is most likely to be written first is the one that corresponds to option B, which is:

Given the logged-in user is on the Change Password page When the user enters the correct current password, enters a valid new password (different from the last 5 passwords), and presses the Change Password button Then the user receives an email confirming that the password has been successfully changed

This ATDD test is most likely to be written first, because it covers the main functionality and the happy path of the user story, as well as the most important acceptance criterion [a]. It also verifies that the user can change the password with a valid new password that meets the criteria related to the length, the type of characters, and the history of the passwords, as specified in the acceptance criterion [c]. The other options are not likely to be written first, because they either cover less critical or less frequent scenarios, such as entering a wrong current password [d] or an invalid new password [e], or they are not related to the user story or the acceptance criteria at all, such as submitting a purchase order [d].

Reference: ISTQB Certified Tester Foundation Level (CTFL) v4.0 sources and documents:

ISTQB® Certified Tester Foundation Level Syllabus v4.0, Chapter 1.3.1, Testing in Software Development Lifecycles1

ISTQB® Glossary of Testing Terms v4.0, Acceptance Test-Driven Development, User Story, Acceptance Criterion, Given-When-Then2

Question #20

Which of the following is not an example of a typical generic skill required for testing?

  • A . Be able to apply test-driven development
  • B . Be able to use test management tools and defect tracking tools
  • C . Be able to communicate defects and failures to developers as objectively as possible
  • D . Possess the necessary social skills that support effective teamwork

Reveal Solution Hide Solution

Correct Answer: A
A

Explanation:

Test-driven development is not an example of a typical generic skill required for testing, but rather an example of a specific technical skill or a development practice that may or may not be relevant for testing, depending on the context and the objectives of the testing activities. Test-driven development is an approach to software development and testing, in which the developers write automated unit tests before writing the source code, and then refactor the code until the tests pass. Test-driven development can help to improve the quality, the design, and the maintainability of the code, as well as to provide fast feedback and guidance for the developers. However, test-driven development is not a skill that is generally expected or needed for testers, especially for testers who are not involved in unit testing or who do not have access to the source code. The other options are examples of typical generic skills required for testing, which are skills that are applicable and beneficial for testing in any context or situation, regardless of the specific testing techniques, tools, or methods used.

The typical generic skills required for testing include:

Be able to use test management tools and defect tracking tools: These are tools that help testers to plan, organize, monitor, and control the testing activities and resources, as well as to record, track, analyze, and resolve the defects detected during testing. These tools can improve the efficiency, the effectiveness, and the communication of the testing process, as well as to provide traceability, metrics, and reports for the testing outcomes.

Be able to communicate defects and failures to developers as objectively as possible: This is a skill that involves the ability to report and describe the defects and failures found during testing in a clear, concise, accurate, and unbiased manner, using relevant information, evidence, and terminology, without making assumptions, judgments, or accusations. This skill can facilitate the collaboration, the understanding, and the resolution of the defects and failures between the testers and the developers, as well as to prevent conflicts, misunderstandings, or blame games.

Possess the necessary social skills that support effective teamwork: These are skills that involve the ability to interact, cooperate, and coordinate with other people involved in or affected by the testing activities, such as the test manager, the test team, the project manager, the developers, the customers, the users, etc. These skills can include communication, negotiation, leadership, motivation, feedback, conflict resolution, etc. These skills can enhance the quality, the productivity, and the satisfaction of the testing process, as well as to foster a positive and constructive testing culture.

Reference: ISTQB Certified Tester Foundation Level (CTFL) v4.0 sources and documents: ISTQB® Certified Tester Foundation Level Syllabus v4.0, Chapter 1.1.1, Testing and the Software Development Lifecycle

ISTQB® Certified Tester Foundation Level Syllabus v4.0, Chapter 1.1.2, Testing and Quality ISTQB® Certified Tester Foundation Level Syllabus v4.0, Chapter 1.2.1, Testing Principles

ISTQB® Certified Tester Foundation Level Syllabus v4.0, Chapter 1.2.2, Testing Policies, Strategies, and Test Approaches

ISTQB® Glossary of Testing Terms v4.0, Test-driven Development, Test Management Tool, Defect Tracking Tool, Defect Report, Failure, Social Skill2

Question #21

Which of the following is a test task that usually occurs during test implementation?

  • A . Make sure the planned test environment is ready to be delivered
  • B . Find, analyze, and remove the causes of the failures highlighted by the tests
  • C . Archive the testware for use in future test projects
  • D . Gather the metrics that are used to guide the test project

Reveal Solution Hide Solution

Correct Answer: A
A

Explanation:

A test task that usually occurs during test implementation is to make sure the planned test environment is ready to be delivered. The test environment is the hardware and software configuration on which the tests are executed, and it should be as close as possible to the production environment where the software system will operate. The test environment should be planned, prepared, and verified before the test execution, to ensure that the test conditions, the test data, the test tools, and the test interfaces are available and functional.

The other options are not test tasks that usually occur during test implementation, but rather test tasks that occur during other test activities, such as:

Find, analyze, and remove the causes of the failures highlighted by the tests: This is a test task that usually occurs during test analysis and design, which is the activity of analyzing the test basis, designing the test cases, and identifying the test data. During this activity, the testers can use techniques such as root cause analysis, defect prevention, or defect analysis, to find, analyze, and remove the causes of the failures highlighted by the previous tests, and to prevent or reduce the occurrence of similar failures in the future tests.

Archive the testware for use in future test projects: This is a test task that usually occurs during test closure, which is the activity of finalizing and reporting the test results, evaluating the test process, and identifying the test improvement actions. During this activity, the testers can archive the testware, which are the test artifacts produced during the testing process, such as the test plan, the test cases, the test data, the test results, the defect reports, etc., for use in future test projects, such as regression testing, maintenance testing, or reuse testing.

Gather the metrics that are used to guide the test project: This is a test task that usually occurs during test monitoring and control, which is the activity of tracking and reviewing the test progress, status, and quality, and taking corrective actions when necessary. During this activity, the testers can gather the metrics, which are the measurements of the testing process, such as the test coverage, the defect density, the test effort, the test duration, etc., that are used to guide the test project, such as planning, estimating, scheduling, reporting, or improving the testing process.

Reference: ISTQB Certified Tester Foundation Level (CTFL) v4.0 sources and documents:

ISTQB® Certified Tester Foundation Level Syllabus v4.0, Chapter 2.1.1, Test Planning1

ISTQB® Certified Tester Foundation Level Syllabus v4.0, Chapter 2.1.2, Test Monitoring and Control1 ISTQB® Certified Tester Foundation Level Syllabus v4.0, Chapter 2.1.3, Test Analysis and Design1 ISTQB® Certified Tester Foundation Level Syllabus v4.0, Chapter 2.1.4, Test Implementation1 ISTQB® Certified Tester Foundation Level Syllabus v4.0, Chapter 2.1.5, Test Execution1 ISTQB® Certified Tester Foundation Level Syllabus v4.0, Chapter 2.1.6, Test Closure1

ISTQB® Glossary of Testing Terms v4.0, Test Environment, Test Condition, Test Data, Test Tool, Test Interface, Failure, Root Cause Analysis, Defect Prevention, Defect Analysis, Testware, Regression Testing, Maintenance Testing, Reuse Testing, Test Coverage, Defect Density, Test Effort, Test Duration2

Question #22

Which of the following statements is true?

  • A . A defect does not always produce a failure, while a bug always produces a failure
  • B . A defect may cause a failure which, when occurring, always causes an error
  • C . Failures can be caused by defects, but also by environmental conditions
  • D . Bugs are defects found during component testing, while failures are defects found at higher test levels

Reveal Solution Hide Solution

Correct Answer: C
C

Explanation:

Failures can be caused by defects, but also by environmental conditions. A failure is an event in which the software system does not perform a required function or performs a function incorrectly, according to the expected behavior. A defect is a flaw in the software system or a deviation from the requirements or the specifications, that may cause a failure. However, not all failures are caused by defects, as some failures may be caused by environmental conditions, such as hardware malfunctions, network interruptions, power outages, incompatible configurations, etc. Environmental conditions are factors that affect the operation of the software system, but are not part of the software system itself.

The other statements are false, because:

A defect does not always produce a failure, while a bug always produces a failure. This statement is false, because a defect may or may not produce a failure, depending on the inputs, the outputs, the states, or the scenarios of the software system, and a bug is just another term for a defect, so it has the same possibility of producing a failure as a defect. For example, a defect in a rarely used feature or a hidden branch of the code may never produce a failure, while a defect in a frequently used feature or a critical path of the code may produce a failure often. A bug is not a different concept from a defect, but rather a synonym or a colloquial term for a defect, so it has the same definition and implications as a defect.

A defect may cause a failure which, when occurring, always causes an error. This statement is false, because an error is not a consequence of a failure, but rather a cause of a defect. An error is a human action or a mistake that produces a defect in the software system, such as a typo, a logic flaw, a requirement misunderstanding, etc. An error is not observable in the software system, but rather in the human mind or the human work products, such as the code, the design, the documentation, etc. A failure is not a cause of an error, but rather a result of a defect, which is a result of an error. For example, an error in the code may cause a defect in the software system, which may cause a failure in the software behavior.

Bugs are defects found during component testing, while failures are defects found at higher test levels. This statement is false, because bugs and failures are not different types of defects, but rather different terms for defects and their manifestations. As mentioned before, bugs are just another word for defects, and failures are the events in which the software system does not perform as expected due to defects. Bugs and failures can be found at any test level, not only at component testing or higher test levels. Test levels are the stages of testing that correspond to the levels of integration of the software system, such as component testing, integration testing, system testing, and acceptance testing. Defects and failures can occur and be detected at any test level, depending on the test objectives, the test basis, the test techniques, and the test environment.

Reference: ISTQB Certified Tester Foundation Level (CTFL) v4.0 sources and documents:

ISTQB® Certified Tester Foundation Level Syllabus v4.0, Chapter 1.1.2, Testing and Quality1 ISTQB® Certified Tester Foundation Level Syllabus v4.0, Chapter 1.2.1, Testing Principles1 ISTQB® Certified Tester Foundation Level Syllabus v4.0, Chapter 1.3.1, Testing in Software Development Lifecycles1

ISTQB® Glossary of Testing Terms v4.0, Failure, Defect, Bug, Environmental Condition, Error, Test Level2

Question #23

Which of the following statements about white-box test techniques is true?

  • A . Achieving full statement coverage and full branch coverage for a software product means that such software product has been fully tested and there are no remaining bugs within the code
  • B . Code-related white-box test techniques are not required to measure the actual code coverage achieved by black-box testing, as code coverage can be measured using the coverage criteria associated with black-box test techniques
  • C . Branch coverage is the most thorough code-related white-box test technique, and therefore applicable standards prescribe achieving full branch coverage at the highest safety levels for safety-critical systems
  • D . Code-related white-box test techniques provide an objective measure of coverage and can be used to complement black-box test techniques to increase confidence in the code

Reveal Solution Hide Solution

Correct Answer: D
D

Explanation:

This answer is correct because code-related white-box test techniques are test design techniques that use the structure of the code to derive test cases. They provide an objective measure of coverage, such as statement coverage, branch coverage, or path coverage, which indicate how much of the code has been exercised by the test cases. Code-related white-box test techniques can be used to complement black-box test techniques, which are test design techniques that use the functional or non-functional requirements of the system or component to derive test cases. By combining both types of techniques, testers can increase their confidence in the code and find more defects.

Reference: ISTQB Glossary of Testing Terms v4.0, ISTQB Foundation Level Syllabus v4.0, Section 2.3.2.2

Question #24

Which of the following statements is true?

  • A . In Agile software development, work product documentation tends to be lightweight and manual tests tend to be often unscripted as they are often produced using experience-based test techniques
  • B . Sequential development models impose the use of systematic test techniques and do not allow the use of experience-based test techniques
  • C . In Agile software development, the first iterations are exclusively dedicated to testing activities, as testing will be used to drive development, which will be performed in the subsequent iterations
  • D . Both in Agile software development and in sequential development models, such as the V-model, test levels tend to overlap since they do not usually have defined entry and exit criteria

Reveal Solution Hide Solution

Correct Answer: A
A

Explanation:

This answer is correct because in Agile software development, work product documentation, such as user stories, acceptance criteria, or test cases, tends to be lightweight and concise, as the focus is on working software and frequent communication rather than comprehensive documentation. Manual tests tend to be often unscripted, as they are often produced using experience-based test techniques, such as error guessing or exploratory testing, which rely on the tester’s skills, knowledge, and creativity to find defects and provide feedback.

Reference: ISTQB Foundation Level Syllabus v4.0, Section 3.1.1.2, Section 3.2.1.2

Question #25

Which of the following statements about the value of maintaining traceability between the test basis and test work products is not true?

  • A . Traceability can be useful for assessing the impact of a change to a test basis item on the corresponding tests
  • B . Traceability can be useful for determining how many test basis items are covered by the corresponding tests
  • C . Traceability can be useful for determining the most suitable test techniques to be used in a testing project
  • D . Traceability can be useful to support the needs required by the auditing of testing

Reveal Solution Hide Solution

Correct Answer: C
C

Explanation:

Traceability is the ability to trace the relationships between the items of the test basis, such as the requirements, the design, the risks, etc., and the test artifacts, such as the test cases, the test results, the defects, etc. Traceability can provide various benefits for the testing process, such as improving the test coverage, the test quality, the test efficiency, and the test communication. However, not all the statements given are true about the value of maintaining traceability between the test basis and test work products. The statement that is not true is option C, which says that test objectives should be the same for all test levels, although the number of tests designed at various levels can vary significantly. This statement is false, because test objectives are the goals or the purposes of testing, which can vary depending on the test level, the test type, the test technique, the test environment, the test stakeholder, etc. Test objectives can be defined in terms of the test basis, the test coverage, the test quality, the test risk, the test cost, the test time, etc. Test objectives should be specific, measurable, achievable, relevant, and time-bound, and they should be aligned with the project objectives and the quality characteristics. Test objectives should not be the same for all test levels, as different test levels have different focuses, scopes, and perspectives of testing, such as component testing, integration testing, system testing, and acceptance testing. The other statements are true about the value of maintaining traceability between the test basis and test work products, such as: Traceability can be useful for assessing the impact of a change to a test basis item on the corresponding tests: This statement is true, because traceability can help to identify which tests are affected by a change in the test basis, such as a new requirement, a modified design, a revised risk, etc., and to determine the necessary actions to update, re-execute, or re-evaluate the tests. Traceability can also help to estimate the effort, the cost, and the time needed to implement the change and to verify its impact on the software system.

Traceability can be useful for determining how many test basis items are covered by the corresponding tests: This statement is true, because traceability can help to measure the test coverage, which is the degree to which the test basis is exercised by the test cases. Traceability can help to identify which test basis items are covered, partially covered, or not covered by the tests, and to evaluate the adequacy, the completeness, and the effectiveness of the testing process. Traceability can also help to identify the gaps, the overlaps, or the redundancies in the test coverage, and to prioritize, optimize, or improve the test cases.

Traceability can be useful to support the needs required by the auditing of testing: This statement is true, because traceability can help to provide evidence, documentation, and justification for the testing activities, results, and outcomes. Traceability can help to demonstrate that the testing process follows the standards, the regulations, the policies, and the best practices that are applicable to the software system, the project, or the organization. Traceability can also help to verify that the testing process meets the expectations, the needs, and the satisfaction of the users and the stakeholders.

Reference: ISTQB Certified Tester Foundation Level (CTFL) v4.0 sources and documents:

ISTQB® Certified Tester Foundation Level Syllabus v4.0, Chapter 1.2.2, Testing Policies, Strategies, and

Test Approaches1

ISTQB® Certified Tester Foundation Level Syllabus v4.0, Chapter 2.1.1, Test Planning1

ISTQB® Certified Tester Foundation Level Syllabus v4.0, Chapter 2.1.2, Test Monitoring and Control1 ISTQB® Certified Tester Foundation Level Syllabus v4.0, Chapter 2.1.3, Test Analysis and Design1 ISTQB® Glossary of Testing Terms v4.0, Traceability, Test Basis, Test Artifact, Test Objective, Test Level, Test Coverage, Test Quality, Test Risk, Test Cost, Test Time2

Question #26

Which of the following statements refers to good testing practice to be applied regardless of the chosen software development model?

  • A . Tests should be written in executable format before the code is written and should act as executable specifications that drive coding
  • B . Test levels should be defined such that the exit criteria of one level are part of the entry criteria for the next level
  • C . Test objectives should be the same for all test levels, although the number of tests designed at various levels can vary significantly
  • D . Involvement of testers in work product reviews should occur as early as possible to take advantage of the early testing principle

Reveal Solution Hide Solution

Correct Answer: D
D

Explanation:

The statement that refers to good testing practice to be applied regardless of the chosen software development model is option D, which says that involvement of testers in work product reviews should occur as early as possible to take advantage of the early testing principle. Work product reviews are static testing techniques, in which the work products of the software development process, such as the requirements, the design, the code, the test cases, etc., are examined by one or more reviewers, with or without the author, to identify defects, violations, or improvements.

Involvement of testers in work product reviews can provide various benefits for the testing process, such as improving the test quality, the test efficiency, and the test communication. The early testing principle states that testing activities should start as early as possible in the software development lifecycle, and should be performed iteratively and continuously throughout the lifecycle. Applying the early testing principle can help to prevent, detect, and remove defects at an early stage, when they are easier, cheaper, and faster to fix, as well as to reduce the risk, the cost, and the time of the testing process. The other options are not good testing practices to be applied regardless of the chosen software development model, but rather specific testing practices that may or may not be applicable or beneficial for testing, depending on the context and the objectives of the testing activities, such as:

Tests should be written in executable format before the code is written and should act as executable specifications that drive coding: This is a specific testing practice that is associated with test-driven development, which is an approach to software development and testing, in which the developers write automated unit tests before writing the source code, and then refactor the code until the tests pass. Test-driven development can help to improve the quality, the design, and the maintainability of the code, as well as to provide fast feedback and guidance for the developers. However, test-driven development is not a good testing practice to be applied regardless of the chosen software development model, as it may not be feasible, suitable, or effective for testing in some contexts or situations, such as when the requirements are unclear, unstable, or complex, when the test automation tools or skills are not available or adequate, when the testing objectives or levels are not aligned with the unit testing, etc.

Test levels should be defined such that the exit criteria of one level are part of the entry criteria for the next level: This is a specific testing practice that is associated with sequential software development models, such as the waterfall model, the V-model, or the W-model, in which the software development and testing activities are performed in a linear and sequential order, with well-defined phases, deliverables, and dependencies. Test levels are the stages of testing that correspond to the levels of integration of the software system, such as component testing, integration testing, system testing, and acceptance testing. Test levels should have clear and measurable entry criteria and exit criteria, which are the conditions that must be met before starting or finishing a test level. In sequential software development models, the exit criteria of one test level are usually part of the entry criteria for the next test level, to ensure that the software system is ready and stable for the next level of testing. However, this is not a good testing practice to be applied regardless of the chosen software development model, as it may not be relevant, flexible, or efficient for testing in some contexts or situations, such as when the software development and testing activities are performed in an iterative and incremental order, with frequent changes, feedback, and adaptations, as in agile software development models, such as Scrum, Kanban, or XP, when the test levels are not clearly defined or distinguished, or when the test levels are performed in parallel or concurrently, etc.

Test objectives should be the same for all test levels, although the number of tests designed at various levels can vary significantly: This is a specific testing practice that is associated with uniform software development models, such as the spiral model, the incremental model, or the prototyping model, in which the software development and testing activities are performed in a cyclical and repetitive manner, with similar phases, deliverables, and processes. Test objectives are the goals or the purposes of testing, which can vary depending on the test level, the test type, the test technique, the test environment, the test stakeholder, etc. Test objectives can be defined in terms of the test basis, the test coverage, the test quality, the test risk, the test cost, the test time, etc. Test objectives should be specific, measurable, achievable, relevant, and time-bound, and they should be aligned with the project objectives and the quality characteristics. In uniform software development models, the test objectives may be the same for all test levels, as the testing process is repeated for each cycle or iteration, with similar focus, scope, and perspective of testing. However, this is not a good testing practice to be applied regardless of the chosen software development model, as it may not be appropriate, realistic, or effective for testing in some contexts or situations, such as when the software development and testing activities are performed in a hierarchical and modular manner, with different phases, deliverables, and dependencies, as in sequential software development models, such as the waterfall model, the V-model, or the W-model, when the test objectives vary according to the test levels, such as component testing, integration testing, system testing, and acceptance testing, or when the test objectives change according to the feedback, the learning, or the adaptation of the testing process, as in agile software development models, such as Scrum, Kanban, or XP, etc.

Reference: ISTQB Certified Tester Foundation Level (CTFL) v4.0 sources and documents:

ISTQB® Certified Tester Foundation Level Syllabus v4.0, Chapter 1.1.1, Testing and the Software Development Lifecycle1

ISTQB® Certified Tester Foundation Level Syllabus v4.0, Chapter 1.2.1, Testing Principles1

ISTQB® Certified Tester Foundation Level Syllabus v4.0, Chapter 1.2.2, Testing Policies, Strategies, and Test Approaches1

ISTQB® Certified Tester Foundation Level Syllabus v4.0, Chapter 1.3.1, Testing in Software Development Lifecycles1

ISTQB® Certified Tester Foundation Level Syllabus v4.0, Chapter 2.1.1, Test Planning1

ISTQB® Certified Tester Foundation Level Syllabus v4.0, Chapter 2.1.2, Test Monitoring and Control1 ISTQB® Certified Tester Foundation Level Syllabus v4.0, Chapter 2.1.3, Test Analysis and Design1 ISTQB® Certified Tester Foundation Level Syllabus v4.0, Chapter 2.1.4, Test Implementation1 ISTQB® Certified Tester Foundation Level Syllabus v4.0, Chapter 2.1.5, Test Execution1 ISTQB® Certified Tester Foundation Level Syllabus v4.0, Chapter 2.1.6, Test Closure1

ISTQB® Glossary of Testing Terms v4.0, Work Product Review, Static Testing, Early Testing, Test-driven Development, Test Level, Entry Criterion, Exit Criterion, Test Objective, Test Basis, Test Coverage, Test Quality, Test Risk, Test Cost, Test Time2

Question #27

A virtual service emulating a real third-party service and the automated test scripts (aimed at testing the system under test) that interact with that service, are test work products that are typically created during:

  • A . Test monitoring and control
  • B . Test implementation
  • C . Test design
  • D . Test analysis

Reveal Solution Hide Solution

Correct Answer: B
B

Explanation:

This answer is correct because test implementation is the activity where test work products, such as test cases, test data, test scripts, test harnesses, test stubs, or virtual services, are created and verified. Test implementation also involves setting up the test environment and preparing the test execution schedule. A virtual service emulating a real third-party service and the automated test scripts that interact with that service are examples of test work products that are typically created during test implementation.

Reference: ISTQB Glossary of Testing Terms v4.0, ISTQB Foundation Level Syllabus v4.0, Section 2.2.2.3

Question #28

The following rules determine the annual bonus to be paid to a salesman of a company based on the total annual amount of the sales made (referred to as TAS).

If the TAS is between 50k and 80k, the bonus is 10%. If the TAS exceeds 80k by a value not greater than 40k, the bonus is 15%. Finally, if the TAS

exceeds the maximum threshold which entitles to a 15% bonus, the bonus is 22%.

Consider applying equivalence partitioning to the TAS (Note: 1k = 1000 euros).

Which one of the following answers contain only test cases that belong to the same equivalence partition?

  • A . TC1 = 81 k; TC2= 97k; TC3=111k; TC4=118k
  • B . TC1 = 40k; TC2= 46k; TC3=51k; TC4=53k
  • C . TC1 = 79k; TC2= 80k; TC3=81k; TC4=82k
  • D . TC1 = 90k; TC2= 110k; TC3=125k: TC4=140k

Reveal Solution Hide Solution

Correct Answer: A
A

Explanation:

This answer is correct because equivalence partitioning is a test design technique that divides the input domain of a system or component into partitions of equivalent data, such that each partition is expected to produce the same output or behavior. Equivalence partitioning aims to reduce the number of test cases by selecting one representative value from each partition. In this case, the input domain of the TAS can be divided into four partitions based on the bonus rules: less than 50k, between 50k and 80k, between 80k and 120k, and more than 120k. The test cases in the answer belong to the same partition, which is between 80k and 120k, and they are expected to produce the same output, which is a bonus of 15%.

Reference: ISTQB Glossary of Testing Terms v4.0, ISTQB Foundation Level Syllabus v4.0, Section 2.3.2.1

Question #29

A typical objective of testing is to ensure that:

  • A . testing is used to drive the development of a software
  • B . a software has been tested using a combination of test techniques
  • C . there are no defects in a software that is about to be released
  • D . a software has been properly covered

Reveal Solution Hide Solution

Correct Answer: B
B

Explanation:

This answer is correct because a typical objective of testing is to ensure that a software has been tested using a combination of test techniques, such as black-box, white-box, or experience-based techniques, that are appropriate for the test objectives, test levels, and test types. Testing using a combination of test techniques can increase the effectiveness and efficiency of testing, as different techniques can target different aspects of the software quality, such as functionality, usability, performance, security, reliability, etc. Testing using a combination of test techniques can also reduce the risk of missing defects that could be detected by one technique but not by another.

Reference: ISTQB Foundation Level Syllabus v4.0, Section 2.3.1.1, Section 2.3.2

Question #30

Consider the following examples of risks identified in different software development projects:

[I]. The contrast color ratio for both normal text and large text of a website does not comply with the applicable accessibility guidelines, making it difficult for many users to read the content on the pages

[II]. A development vendor fails to deliver their software system on time, causing significant delays to system integration testing activities that have been planned as part of a development project for a system of systems

[III]. People in the test team do not have sufficient skills to automate tests at the test levels required by the test automation strategy which does not allow production of an effective regression test suite [IV]. In a web application, data from untrusted sources is not subject to proper input validation, making the application vulnerable to several security attacks

Which of the following statements is true?

  • A . [I] and [III] are product risks; [II] and [IV] are project risks
  • B . [I] and [IV] are product risks. [II] and [III] are project risks
  • C . [II], [III] and [IV] are product risks; [I] is a project risk
  • D . [IV] is a product risk; [I]. [II] and [III] are project risks

Reveal Solution Hide Solution

Correct Answer: B
B

Explanation:

This answer is correct because product risks are risks that affect the quality of the software product, such as defects, failures, or non-compliance with requirements or standards. Project risks are risks that affect the project’s schedule, budget, resources, or scope, such as delays, cost overruns, skill gaps, or scope changes. In this case, [I] and [IV] are product risks, as they relate to the accessibility and security of the software product, which are quality attributes. [II] and [III] are project risks, as they relate to the delivery time and the test automation skills of the test team, which are project factors.

Reference: ISTQB Glossary of Testing Terms v4.0, ISTQB Foundation Level Syllabus v4.0, Section 2.1.1.1

Question #31

Following a risk-based testing approach you have designed 10 tests to cover a product risk with a high-risk level. You want to estimate, adopting the three-point test estimation technique, the test effort required to reduce the risk level to zero by executing those 10 tests.

You made the following three initial estimates:

• most optimistic = 6 person hours

• most likely = 30 person hours

• most pessimistic = 54 person hours

Based only on the given information, which of the following answers about the three-point test

estimation technique applied to this problem is true?

  • A . The final estimate is between 22 person hours and 38 person hours
  • B . The final estimate is exactly 30 person hours because the technique uses the initial most likely estimate as the final estimate
  • C . The final estimate is between 6 person hours and 54 person hours
  • D . The final estimate is exactly 30 person hours because the technique uses the arithmetic mean of the three initial estimates as the final estimate

Reveal Solution Hide Solution

Correct Answer: A
A

Explanation:

The three-point test estimation technique is a method of estimating the test effort based on three initial estimates: the most optimistic, the most likely, and the most pessimistic. The technique uses a weighted average of these three estimates to calculate the final estimate, which is also known as the expected value.

The formula for the expected value is:

Expected value = (most optimistic + 4 * most likely + most pessimistic) / 6

Using the given values, the expected value is:

Expected value = (6 + 4 * 30 + 54) / 6 Expected value = 30 person hours

However, the expected value is not the only factor to consider when estimating the test effort. The technique also calculates the standard deviation, which is a measure of the variability or uncertainty of the estimates. The formula for the standard deviation is: Standard deviation = (most pessimistic – most optimistic) / 6

Using the given values, the standard deviation is:

Standard deviation = (54 – 6) / 6 Standard deviation = 8 person hours

The standard deviation can be used to determine a range of possible values for the test effort, based on a certain level of confidence. For example, using a 68% confidence level, the range is: Expected value ± standard deviation

Using the calculated values, the range is:

30 ± 8 person hours

Therefore, the final estimate is between 22 person hours and 38 person hours, which is option A.

Reference: ISTQB® Certified Tester Foundation Level Syllabus v4.01, Section 2.3.2, page 24-25; ISTQB® Glossary v4.02, page 33.

Question #32

The acceptance criteria associated with a user story:

  • A . are often written in a rule-oriented format using the template referred to as "Given/When/Then"
  • B . are often documented following in rule-oriented format using the following template: "As a [role], I want [feature], so that I can [benefit]"
  • C . can be written in different formats and represent an aspect of a user story referred to as confirmation’ of the so called "3 C’s"
  • D . must be written in one of the two following formats: scenario-oriented or rule-oriented

Reveal Solution Hide Solution

Correct Answer: C
C

Explanation:

The acceptance criteria associated with a user story are the conditions that must be met for the user story to be considered done and to deliver the expected value to the user. They are often written in different formats, such as rule-oriented, scenario-oriented, or table-oriented, depending on the nature and complexity of the user story. They represent an aspect of a user story referred to as confirmation, which is one of the so called “3 C’s” of user stories. The other two aspects are card and conversation. Card refers to the concise and informal description of the user story, usually following the template: “As a [role], I want [feature], so that I can [benefit]”. Conversation refers to the ongoing dialogue between the stakeholders and the team members to clarify and refine the user story and its acceptance criteria. Therefore, option C is the correct answer.

Reference: ISTQB® Certified Tester Foundation Level Syllabus v4.01, Section 3.2.2, page 35-36; ISTQB® Glossary v4.02, page 37.

Question #33

Consider a review for a high-level architectural document written by a software architect. The architect does most of the review preparation work, including distributing the document to reviewers before the review meeting. However, reviewers are not required to analyze the document in advance, and during the review meeting the software architect explains the document step by step. The only goal of this review is to establish a common understanding of the software architecture that will be used in a software development project.

Which of the following review types does this review refer to?

  • A . Inspection
  • B . Audit
  • C . Walkthrough
  • D . Informal review

Reveal Solution Hide Solution

Correct Answer: C
C

Explanation:

This answer is correct because a walkthrough is a type of review where the author of the work product leads the review process and explains the work product to the reviewers. The reviewers are not required to prepare for the review in advance, and the main objective of the walkthrough is to establish a common understanding of the work product and to identify any major defects or issues. A walkthrough is usually informal and does not follow a defined process or roles. In this case, the review for a high-level architectural document written by a software architect matches the characteristics of a walkthrough.

Reference: ISTQB Glossary of Testing Terms v4.0, ISTQB Foundation Level Syllabus v4.0, Section 2.4.2.2

Question #34

Which of the following statements about the shift-left approach is true?

  • A . Shift-left in testing can be implemented only in Agile/DevOps frameworks, as it relies completely on automated testing activities performed within a continuous integration process
  • B . Performance testing performed during component testing, is a form of shift-left in testing that avoids planning and executing costly end-to-end testing at the system test level in a production-like environment
  • C . Shift-left in testing can be implemented in several ways to find functional defects early in the lifecycle, but it cannot be relied upon to find defects associated with non-functional characteristics
  • D . Continuous integration supports shift-left in testing as it can reduce the time between the introduction of a defect and its detection, thereby reducing the cost to fix it

Reveal Solution Hide Solution

Correct Answer: D
D

Explanation:

This answer is correct because shift-left in testing is an approach that aims to perform testing activities as early as possible in the software development lifecycle, in order to find and fix defects faster and cheaper, and to improve the quality of the software product. Continuous integration is a practice that supports shift-left in testing, as it involves integrating and testing the software components frequently, usually several times a day, using automated tools and processes. Continuous integration can reduce the time between the introduction of a defect and its detection, thereby reducing the cost to fix it and the risk of accumulating defects that could affect the functionality or performance of the software product.

Reference: ISTQB Foundation Level Syllabus v4.0, Section 3.1.1.3, Section 3.2.1.3

Question #35

The fact that defects are usually not evenly distributed among the various modules that make up a software application, but rather their distribution tend to reflect the Pareto principle:

  • A . is a false myth
  • B . is expressed by the testing principle referred to as Tests wear out’
  • C . is expressed by the testing principle referred to as ‘Defects cluster together’
  • D . is expressed by the testing principle referred to as ‘Bug prediction’

Reveal Solution Hide Solution

Correct Answer: C
C

Explanation:

The fact that defects are usually not evenly distributed among the various modules that make up a software application, but rather their distribution tend to reflect the Pareto principle, is expressed by the testing principle referred to as ‘Defects cluster together’. This principle states that a small number of modules contain most of the defects detected, or that a small number of causes are responsible for most of the defects. This principle can be used to guide the test analysis and design activities, by prioritizing the testing of the most critical or risky modules, or by applying more rigorous test techniques to them. Therefore, option C is the correct answer.

Reference: ISTQB® Certified Tester Foundation Level Syllabus v4.01, Section 1.2.1, page 11; ISTQB® Glossary v4.02, page 16.

Question #36

Which of the following statements is true?

  • A . Functional testing focuses on what the system should do while non-functional testing on the internal structure of the system
  • B . Non-functional testing includes testing of both technical and non-technical quality characteristics
  • C . Testers who perform functional tests are generally expected to have more technical skills than testers who perform non-functional tests
  • D . The test techniques that can be used to design white-box tests are described in the ISO/IEC 25010 standard

Reveal Solution Hide Solution

Correct Answer: B
B

Explanation:

Non-functional testing includes testing of both technical and non-technical quality characteristics. Non-functional testing is the process of testing the quality attributes of a system, such as performance, usability, security, reliability, etc. Non-functional testing can be applied at any test level and can use both black-box and white-box test techniques. Non-functional testing can cover both technical aspects, such as response time, throughput, resource consumption, etc., and non-technical aspects, such as user satisfaction, accessibility, compliance, etc. Therefore, option B is the correct answer.

Reference: ISTQB® Certified Tester Foundation Level Syllabus v4.01, Section 1.3.1, page 13; ISTQB® Glossary v4.02, page 40.

Question #37

Which of the following work products cannot be examined by static analysis?

  • A . Test plans
  • B . Source code
  • C . Compiled code
  • D . Formal models

Reveal Solution Hide Solution

Correct Answer: A
A

Explanation:

Static analysis is the process of examining the work products of a software development or testing activity without executing them. Static analysis can be applied to various types of work products, such as requirements, design, code, test cases, etc. However, test plans are not suitable for static analysis, because they are high-level documents that describe the test objectives, scope, strategy, resources, schedule, and risks of a testing project. Test plans are not executable or formalized in a way that static analysis tools can analyze them. Therefore, option A is the correct answer.

Reference: ISTQB® Certified Tester Foundation Level Syllabus v4.01, Section 2.2.1, page 20; ISTQB® Glossary v4.02, page 45.

Question #38

Confirmation testing is performed after:

  • A . a defect is fixed and after other tests do not find any side-effect introduced in the software as a result of such fix
  • B . a failed test, and aims to run that test again to confirm that the same behavior still occurs and thus appears to be reproducible
  • C . the execution of an automated regression test suite to confirm the absence of false positives in the test results
  • D . a defect is fixed, and if such testing is successful then the regression tests that are relevant for such fix can be executed

Reveal Solution Hide Solution

Correct Answer: D
D

Explanation:

Confirmation testing is performed after a defect is fixed, and if such testing is successful then the regression tests that are relevant for such fix can be executed. Confirmation testing, also known as re-testing, is the process of verifying that a defect has been resolved by running the test case that originally detected the defect. Confirmation testing is usually done before regression testing, which is the process of verifying that no new defects have been introduced in the software as a result of changes or fixes. Therefore, option D is the correct answer.

Reference: ISTQB® Certified Tester Foundation Level Syllabus v4.01, Section 2.4.1, page 28; ISTQB® Glossary v4.02, page 15.

Question #39

Which of the following statements about static testing and dynamic testing is true?

  • A . Unlike dynamic testing, which can be also performed manually, static testing cannot be performed without specialized tools
  • B . Static testing is usually much less cost-effective than dynamic testing
  • C . Unlike dynamic testing, which focuses on detecting potential defects, static testing focuses on detecting failures which may be due to actual defects
  • D . Both static testing and dynamic testing can be used to highlight issues associated with non-functional characteristics

Reveal Solution Hide Solution

Correct Answer: D
D

Explanation:

This answer is correct because static testing and dynamic testing are both types of testing that can be used to highlight issues associated with non-functional characteristics, such as usability, performance, security, reliability, etc. Static testing is a type of testing that involves the analysis of software work products, such as requirements, design, code, or test cases, without executing them. Dynamic testing is a type of testing that involves the execution of software work products, such as code or test cases, using inputs and verifying outputs. Both static testing and dynamic testing can be applied to different test levels and test types, and can use different test techniques and tools, to evaluate the non-functional characteristics of the software product.

Reference: ISTQB Glossary of Testing Terms v4.0, ISTQB Foundation Level Syllabus v4.0, Section 2.2.1.1, Section 2.2.1.2

Question #40

Which of the following is a test-first approach, where tests that express a shared understanding from stakeholders of how the application is expected to work, are first written in business-readable language (following the Given/When/Then format), and then made executable to drive development?

  • A . Test-Driven Development (TDD)
  • B . Acceptance Test-Driven Development (ATDD)
  • C . Behavior-Driven Development (BDD)
  • D . Domain-Driven Design (DDD)

Reveal Solution Hide Solution

Correct Answer: C
C

Explanation:

This answer is correct because Behavior-Driven Development (BDD) is a test-first approach, where tests that express a shared understanding from stakeholders of how the application is expected to work, are first written in business-readable language (following the Given/When/Then format), and then made executable to drive development. BDD is a collaborative approach that involves testers, developers, business analysts, product owners, and other stakeholders in defining the expected behavior of the application using scenarios that describe the preconditions, actions, and outcomes of the application. BDD scenarios are written using a domain-specific language (DSL) that can be translated into executable test cases using tools such as Cucumber or SpecFlow. BDD aims to improve communication, collaboration, and feedback among the team members, and to deliver software that meets the customer’s needs and expectations.

Reference: ISTQB Glossary of Testing Terms v4.0, ISTQB Foundation Level Syllabus v4.0, Section 3.1.1.4

Question #41

Which of the following statements is not correct?

  • A . Looking for defects in a system may require Ignoring system details
  • B . Identifying defects may be perceived as criticism against product
  • C . Looking for defects in system requires professional pessimism and curiosity
  • D . Testing is often seen as a destructive activity instead of constructive activity

Reveal Solution Hide Solution

Correct Answer: A
A

Explanation:

Looking for defects in a system does not require ignoring system details, but rather paying attention to them and understanding how they affect the system’s quality, functionality, and usability. Ignoring system details could lead to missing important defects or testing irrelevant aspects of the system.

Identifying defects may be perceived as criticism against product, especially by the developers or stakeholders who are invested in the product’s success. However, identifying defects is not meant to be a personal attack, but rather a constructive feedback that helps to improve the product and ensure its alignment with the requirements and expectations of the users and clients. Looking for defects in system requires professional pessimism and curiosity, as testers need to anticipate and explore the possible ways that the system could fail, malfunction, or behave unexpectedly. Professional pessimism means being skeptical and critical of the system’s quality and reliability, while curiosity means being eager and interested in finding out the root causes and consequences of the defects.

Testing is often seen as a destructive activity instead of constructive activity, as it involves finding and reporting the flaws and weaknesses of the system, rather than creating or enhancing it. However, testing is actually a constructive activity, as it contributes to the system’s improvement, verification, validation, and optimization, and ultimately to the delivery of a high-quality product that meets the needs and expectations of the users and clients.

Question #42

Which of the following s the most correct statement about state testing techniques?

  • A . Static techniques can be used before all code is ready for execution
  • B . Static techniques find more detects then dynamic techniques.
  • C . Static techniques can be used by inexperienced users.
  • D . Static techniques are always cheaper than dynamic techniques.

Reveal Solution Hide Solution

Correct Answer: A
A

Explanation:

State testing techniques are a type of dynamic testing techniques that are based on the behavior of the system under test for different input conditions and events. Dynamic testing techniques require the system to be executed with test cases, whereas static testing techniques do not. Static testing techniques can be applied before the code is ready for execution, such as reviews, inspections, walkthroughs, and static analysis. Static testing techniques can help find defects early in the development process, improve the quality of the code, and reduce the cost and effort of dynamic testing. Reference = ISTQB Certified Tester Foundation Level (CTFL) v4.0 Syllabus, Chapter 4, Section 4.2.1, Page 281; ISTQB Glossary of Testing Terms v4.0, Page 292

Question #43

A test manager decided to skip static testing since he believes bugs can be found easily by doing dynamic testing. Was this decision right or wrong?

  • A . The decision was wrong. Ensuring quality mandates that static testing is performed after performing the dynamic testing.
  • B . The decision was right. Static testing is usually redundant if a product is planned to go through a full-cycle of dynamic testing.
  • C . The decision was right. Most of the bugs are easier to identify during the dynamic testing.
  • D . The decision was wrong. Static testing can find defects early in the development process, reducing the overall cost of testing and development

Reveal Solution Hide Solution

Correct Answer: D
D

Explanation:

Static testing is a form of testing that does not involve executing the software or system under test. It includes activities such as reviews, inspections, walkthroughs, and analysis of documents, code, and models. Static testing can find defects early in the development process, before they become more expensive and difficult to fix in later stages. Static testing can also improve the quality of the software or system by preventing defects from being introduced in the first place. Static testing can complement dynamic testing, which involves executing the software or system under test and checking the results against expected outcomes. Dynamic testing can find defects that static testing may miss, such as performance, usability, or integration issues. However, dynamic testing alone is not sufficient to ensure quality, as it may not cover all possible scenarios, inputs, or paths. Therefore, a test manager who decides to skip static testing is making a wrong decision, as he or she is ignoring the benefits of static testing and relying solely on dynamic testing, which may not be effective or efficient enough to find and prevent defects. Reference = ISTQB Certified Tester Foundation Level Syllabus, Version 4.0, 2018, Section 2.1.1, page 14; ISTQB Glossary of Testing Terms, Version 4.0, 2018, page 36; ISTQB CTFL 4.0 – Sample Exam – Answers, Version 1.1, 2023, Question 3, page 9.

Question #44

In which of the following test documents would you expect to find test exit criteria described9

  • A . Test design specification
  • B . Project plan
  • C . Requirements specification
  • D . Test plan

Reveal Solution Hide Solution

Correct Answer: D
D

Explanation:

Test exit criteria are the conditions that must be fulfilled before concluding a particular testing phase. These criteria act as a checkpoint to assess whether we have achieved the testing objectives and are done with testing1. Test exit criteria are typically defined in the test plan document, which is one of the outputs of the test planning phase. The test plan document describes the scope, approach, resources, and schedule of the testing activities. It also identifies the test items, the features to be tested, the testing tasks, the risks, and the test deliverables2. According to the ISTQB® Certified Tester Foundation Level Syllabus v4.0, the test plan document should include the following information related to the test exit criteria3:

The criteria for evaluating test completion, such as the percentage of test cases executed, the percentage of test coverage achieved, the number and severity of defects found and fixed, the quality and reliability of the software product, and the stakeholder satisfaction.

The criteria for evaluating test process improvement, such as the adherence to the test strategy, the efficiency and effectiveness of the testing activities, the lessons learned and best practices identified, and the recommendations for future improvements.

Therefore, the test plan document is the most appropriate test document to find the test exit criteria described. The other options, such as test design specification, project plan, and requirements specification, are not directly related to the test exit criteria. The test design specification describes the test cases and test procedures for a specific test level or test type3. The project plan describes the overall objectives, scope, assumptions, risks, and deliverables of the software project4. The requirements specification describes the functional and non-functional requirements of the software product5. None of these documents specify the conditions for ending the testing process or evaluating the testing outcomes. Reference = ISTQB® Certified Tester Foundation Level Syllabus v4.0, Entry and Exit Criteria in Software Testing | Baeldung on Computer Science, Entry And Exit Criteria In Software Testing – Rishabh Software, Entry and Exit Criteria in Software Testing Life Cycle – STLC [2022 Updated] – Testsigma Blog, ISTQB® releases Certified Tester Foundation Level v4.0 (CTFL).

Question #45

The following 4 equivalence classes are given:

Which of the following alternatives includes correct test values for x. based on equivalence partitioning?

  • A . -100; 100:1000; 1001
  • B . -500; 0; 100; 1000
  • C . -99; 99:101; 1001
  • D . -1000; -100; 100; 1000

Reveal Solution Hide Solution

Correct Answer: D
D

Explanation:

The question is about selecting the correct test values for x based on equivalence partitioning.

Equivalence partitioning is a software test design technique that divides the input data of a software unit into partitions of equivalent data from which test cases can be derived.

In this case, the given equivalence classes are:

(x leq -100)

(-100 < x < 100)

(100 leq x < 1000)

(x geq 1000)

Option D provides a value from each of these partitions:

For (x leq -100), it gives -1000.

For (-100 < x < 100), it gives -100 and 100.

For (100 leq x < 1000), it gives 500.

For (x geq 1000), it gives 1500.

So, option D covers all four given equivalence classes with appropriate values.

Reference: ISTQB Certified Tester Foundation Level (CTFL) v4.0 documents available at ISTQB and ASTQB.

1: ISTQB Foundation Level Syllabus 2018, Version 4.0, p. 38

2: ISTQB Foundation Level Syllabus 2018, Version 4.0, p. 39

: ISTQB Foundation Level Syllabus 2018, Version 4.0, p. 40

Question #46

Which of the following coverage criteria results in the highest coverage for state transition based test cases?

  • A . Can’t be determined
  • B . Covering all transitions at least once
  • C . Covering only start and end states
  • D . Covering all states at least once

Reveal Solution Hide Solution

Correct Answer: B
B

Explanation:

Covering all transitions at least once is the highest coverage criterion for state transition based test cases, because it ensures that every possible change of state is tested at least once. This means that all the events that trigger the transitions, as well as the actions and outputs that result from the transitions, are verified. Covering all transitions at least once also implies covering all states at least once, but not vice versa. Therefore, option D is not the highest coverage criterion. Option C is the lowest coverage criterion, because it only tests the initial and final states of the system or component, without checking the intermediate states or transitions. Option A is incorrect, because the coverage criteria for state transition based test cases can be determined and compared based on the number of transitions and states covered. Reference = CTFL 4.0 Syllabus, Section 4.2.3, page 49-50.

Question #47

Which of the following lists factors That contribute to PROJECT risks?

  • A . skill and staff shortages; problems in defining the right requirements, contractual issues.
  • B . skill and staff shortages; software does not perform its intended functions; problems in defining the right requirements.
  • C . problems in defining the right requirements; contractual issues; poor software quality characteristics.
  • D . poor software quality characteristics; software does not perform its intended functions.

Reveal Solution Hide Solution

Correct Answer: A
A

Explanation:

Project risks are the uncertainties or threats that may affect the project objectives, such as scope, schedule, cost, and quality. According to the ISTQB Certified Tester Foundation Level (CTFL) v4.0 syllabus, some of the factors that contribute to project risks are:

Skill and staff shortages: This factor refers to the lack of adequate or qualified human resources to perform the project tasks. This may result in delays, errors, rework, or low productivity. Problems in defining the right requirements: This factor refers to the difficulties or ambiguities in eliciting, analyzing, specifying, validating, or managing the requirements of the project. This may result in misalignment, inconsistencies, gaps, or changes in the requirements, affecting the project scope and quality.

Contractual issues: This factor refers to the challenges or disputes that may arise from the contractual agreements between the project parties, such as clients, suppliers, vendors, or subcontractors. This may result in legal, financial, or ethical risks, affecting the project delivery and satisfaction.

The other options are not correct because they list factors that contribute to PRODUCT risks, not project risks. Product risks are the uncertainties or threats that may affect the quality or functionality of the software product or system.

Some of the factors that contribute to product risks are:

Poor software quality characteristics: This factor refers to the lack of adherence or compliance to the quality attributes or criteria of the software product or system, such as reliability, usability, security, performance, or maintainability. This may result in defects, failures, or dissatisfaction of the users or stakeholders.

Software does not perform its intended functions: This factor refers to the deviation or discrepancy between the expected and actual behavior or output of the software product or system. This may result in errors, faults, or malfunctions of the software product or system.

Reference = ISTQB Certified Tester Foundation Level (CTFL) v4.0 syllabus, Chapter 1: Fundamentals of Testing, Section 1.5: Risks and Testing, Pages 14-16.

Question #48

Which sequence of stated in the answer choices is correct in accordance with the following figure depicting the life-cycle of a defect?

  • A . S0->S1->S2->S3->S5->S1
  • B . S0->S1->S2->S3->S5->S1->S2->S3
  • C . S0->S1->S2~>S3->S4
  • D . S0->S1 ->S2->S3->S5->S3->S4

Reveal Solution Hide Solution

Correct Answer: D
D

Explanation:

According to the ISTQB Certified Tester Foundation Level (CTFL) v4.0, the life cycle of a defect typically follows a sequence from its discovery to its closure. In the provided figure, it starts with S0 (New), moves to S1 (Assigned), then to S2 (Resolved), followed by S3 (Verified). If the defect is not fixed, it can be Re-opened (S5) and goes back for verification (S3). Once verified, it is Closed (S4).

Reference: ISTQB Certified Tester Foundation Level (CTFL) v4.0 Syllabus, Section 1.4.3, Page 17.

Question #49

A Test Manager conducts risk assessment for a project. One of the identified risks is: The sub-contractor may fail to meet his commitment". If this risk materializes. it will lead to delay in completion of testing required for the current cycle.

Which of the following sentences correctly describes the risk?

  • A . It is a product risk since any risk associated with development timeline is a product risk.
  • B . It is no longer a risk for the Test Manager since an independent party (the sub-contractor) is now managing it
  • C . It is a object risk since successful completion of the object depends on successful and timely completion of the tests
  • D . It is a product risk since default on part of the sub-contractor may lead to delay in release of the product

Reveal Solution Hide Solution

Correct Answer: D
D

Explanation:

A product risk is a risk that affects the quality or timeliness of the software product being developed or tested1. Product risks are related to the requirements, design, implementation, verification, and maintenance of the software product2.

The risk of the sub-contractor failing to meet his commitment is a product risk, as it could cause a delay in the completion of the testing required for the current cycle, which in turn could affect the release date of the product. The release date is an important aspect of the product quality, as it reflects the customer satisfaction and the market competitiveness of the product3. The other options are not correct because:

Question #50

Who of the following has the best knowledge to decide what tests in a test project should be automated?

  • A . The developer
  • B . The customer
  • C . The development manager
  • D . The test leader

Reveal Solution Hide Solution

Correct Answer: D
D

Explanation:

The test leader is the person who is responsible for planning, monitoring, and controlling the test activities and resources in a test project. The test leader should have the best knowledge of the test objectives, scope, risks, resources, schedule, and quality criteria. The test leader should also be aware of the test automation criteria, such as the execution frequency, the test support, the team education, the roles and responsibilities, and the devs and testers collaboration1. Based on these factors, the test leader can decide which tests are suitable for automation and which are not, and prioritize them accordingly. The test leader can also coordinate with the test automation engineers, the developers, and the stakeholders to ensure the alignment of the test automation strategy with the test project goals and expectations. Reference = ISTQB Certified Tester Foundation Level (CTFL) v4.0 Syllabus, Chapter 2, Section 2.3.1, Page 152; ISTQB Glossary of Testing Terms v4.0, Page 403; ISTQB Certified Tester Foundation Level (CTFL) v4.0 Syllabus, Chapter 6, Section 6.1.1, Page 514; Top 8 Test Automation Criteria You Need To Fulfill – QAMIND1

Exit mobile version